La razón principal se debe a que el prisma refracta la luz de tal manera que la parte "azul" se extiende más que la parte "roja". De modo que, en general, la energía que incide en el termómetro es mayor en la parte infrarroja y roja que en la parte azul del espectro.
Edición: Acabo de ver tu edición. Tienes razón. Allí se puede ver el detalles.
Esta es una cita del sitio web:
La respuesta resulta ser el diseño experimental, y un fallo en la corrección de la refracción. En el montaje de Herschel, la luz solar es refractada por un prisma. El índice de refracción, por supuesto, debe variar con la longitud de onda para que la luz solar se disperse en sus distintos colores. Si el índice de refracción variara linealmente con la longitud de onda, Herschel no habría necesitado corregir esa variación, ya que las longitudes de onda estarían uniformemente espaciadas a lo largo de su mesa. Sin embargo, como el índice de refracción varía de forma no lineal con la longitud de onda, las longitudes de onda no estarán espaciadas uniformemente a lo largo de la mesa de medición de Herschel. El espaciamiento real de las longitudes de onda frente a la distancia a lo largo de su mesa para un ángulo de incidencia de 45° del aire en el vidrio muestra que la región infrarroja está mucho más concentrada que las longitudes de onda ópticas. (El gráfico muestra el espaciado a lo largo del espectro dividido por la distancia desde el prisma. Por lo tanto, para obtener el espaciamiento real en cm o pulgadas, multiplique por la distancia desde el prisma en cm o pulgadas). El factor de concentración relativa se muestra normalizado a 0,60 micras. El resultado neto es que la "temperatura" observada por Herschel debería alcanzar un pico en el infrarrojo.
El sitio web también ofrece algunos gráficos e in un se puede ver que la parte roja (~650 nm) del espectro es unas 3 veces más concentrada que la azul (~450 nm), mientras que la parte infrarroja es aún más concentrada.
4 votos
Creo que debería hacer su puesto autocontenido Es decir, explique el montaje experimental que está considerando para que no tengamos que seguir un enlace, sólo para poder entender su pregunta.
1 votos
Ver physics.stackexchange.com/questions/59456/ .